Phase portrait of nonlinear system of differential equations

Click For Summary
The phase portrait of the nonlinear system defined by x' = x^2 and y' = y^2 features an equilibrium point at (0,0) and consists of equilibria along the axes x = 0 and y = 0. The associated linearized system is x' = 0, y' = 0, indicating stability at the origin. Solutions for the nonlinear system are found to be x(t) = -1/(t + C1) and y(t) = -1/(t + C2), which can be expressed in terms of initial conditions. In the case where C1 = C2 = 0, trajectories follow the line y = x, with points approaching the origin from different quadrants based on the sign of t. Overall, all trajectories appear to converge towards the origin along straight lines.
p3forlife
Messages
20
Reaction score
0

Homework Statement



Describe the phase portrait of the nonlinear system x' = x^2, y' = y^2
Also, find the equilibrium points and describe the behaviour of the associated linearized system.

The Attempt at a Solution



We have an equilibrium point at (0,0).
The associated linearized system is x' = 0, y' = 0. The phase portrait for this consists of lines of equilibria along x = 0, and y = 0.

For the nonlinear system, I have found solutions x(t) = -1/t and y(t) = -1/t. I don't know what these solutions mean in terms of a phase portrait. Nor can I express the solutions in terms of constants x_0 and y_0.
 
Physics news on Phys.org
It looks like you have left off the constant of integration when you found solutions x(t) and y(t). Both of your differential equations are separable.

dx/dt = x2 ==> dx/x2 = dt ==> \int dx/x^2 = \int dt
==> -1/x = t + C1 ==> x = -1/(t + C1)

Similarly, y = -1/(t + C2)
You should be able to determine the constant from your initial conditions.

In the special case where C1 = C2 = 0, the trajectories follow the line y = x. If t > 0, the solution points approach the origin along the part of the line in the third quadrant. If t < 0, the solution points approach the origin along the part of the line in the first quadrant. Different initial conditions will generate different trajectories, but I believe all of them will be straight lines pointing into the origin.

Does that make sense? It has been a lot of years since I studied dynamical systems, so I might be a little off base on some of this.
 
Thanks :) It makes sense.
 
Question: A clock's minute hand has length 4 and its hour hand has length 3. What is the distance between the tips at the moment when it is increasing most rapidly?(Putnam Exam Question) Answer: Making assumption that both the hands moves at constant angular velocities, the answer is ## \sqrt{7} .## But don't you think this assumption is somewhat doubtful and wrong?

Similar threads

  • · Replies 1 ·
Replies
1
Views
2K
Replies
3
Views
2K
  • · Replies 4 ·
Replies
4
Views
2K
Replies
3
Views
2K
Replies
2
Views
1K
  • · Replies 27 ·
Replies
27
Views
2K
Replies
2
Views
1K
Replies
10
Views
2K
  • · Replies 1 ·
Replies
1
Views
2K
  • · Replies 7 ·
Replies
7
Views
2K